Изменения

Перейти к: навигация, поиск

1ripippmtnsumwu

2013 байт добавлено, 20:06, 5 июня 2016
Динамика
Докажем неравенство <tex>W_k(t_u, t_v, m) \leqslant \max(W_{k-1}(t_u, t_v, m), W'_k)</tex>. Положим <tex>Z</tex> реализует <tex> W_k(t_u, t_v, m)</tex>.
* <tex>J_k \notin Z</tex>. *:Тогда <tex>W_k(t_u, t_v, m) = W_{k-1}(t_u, t_v, m) \leqslant \max(W_{k-1}(t_u, t_v, m), W'_k)</tex>.* <tex>J_k \in Z</tex>. *:Положим <tex>t_x</tex> и <tex>t_y</tex> временем начала выполнения и завершения работы <tex>J_k</tex> в <tex>JPS</tex> от <tex>Z</tex>. Благодаря предыдущей лемме, мы знаем, что <tex>t_x, t_y \in \Theta</tex>. Также выполняется условие <tex> \max(r_k, t_u) \leqslant t_x < t_y \leqslant \min(d_k, t_v).</tex> Пусть <tex>Z_1, Z_2, Z_3</tex> будут подмножествами <tex>Z/J_k</tex> такими, что все работы в <tex>Z_1, Z_2</tex> и <tex>Z_3</tex> имеют время появления <tex>r_i</tex> в границах <tex>[t_u,t_x)</tex>, <tex>[t_x, t_y)</tex> и <tex>[t_y,t_v)</tex> соответственно. По структуре <tex>JPS</tex>(работа <tex>J_k</tex> имеет максимальный дедлайн <tex>d_k</tex>) все работы в <tex>Z_1</tex> завершаться до <tex>t_x</tex>. Более того, все работы в <tex>Z_2</tex> начнут выполняться после <tex>t_x</tex> и завершаться до <tex>t_y</tex>, аналогично для <tex>Z_3</tex>. Также <tex>p \cdot (|Z_2| + 1) = t_y - t_x,</tex> так как <tex>J_k</tex> выполнялась в промежутке <tex>[t_x, t_y).</tex> При этом, <tex>|Z_1| + |Z_2| + |Z_3| = m - 1.</tex> Можно заметить, что <tex>weight(Z_1) \leqslant W_{k-1}(t_u, t_x, |Z_1|), weight(Z_2) \leqslant W_{k-2}(t_x, t_y, |Z_2|)</tex> и <tex>weight(Z_3) \leqslant W_{k-1}(t_y, t_v, |Z_3|).</tex> Следовательно, <tex>W_k(t_u, t_v, m) = w_k + \sum\limits_{i = 1}^3weight(Z_i) \leqslant W'_k \leqslant \max(W_{k-1}(t_u, t_v, m), W'_k).</tex> Исходя из двух неравенств доказанных выше, можно получить требуемое равенство <tex>W_k(t_u, t_v, m) = \max(W_{k-1}(t_u, t_v, m), W'_k).</tex> Лемма доказана.
}}
32
правки

Навигация